dan
Thanks Received: 155
Atticus Finch
Atticus Finch
 
Posts: 202
Joined: March 10th, 2009
 
 
 

Q17 - When exercising the muscles

by dan Fri Dec 31, 1999 8:00 pm

17. (B)
Question Type: Assumption

The argument seems reasonable, but consider someone who may, for whatever reason, have unequal strength on either side of his spine to begin with. Wouldn’t he want to exercise the weaker side more than the stronger side in order to make them equal? The assumption in the argument is that unequal exercise leads to unequal muscle strength (in fact this may not be true for all people). Answer (B) clearly expresses this assumption.

(A) is incorrect. The argument does not need to assume that equal muscle strength will be enough on its own to keep the spine aligned (other things might be necessary as well). The argument says that equal muscle strength is one thing that is required to keep the spine aligned, but it doesn’t have to be enough on its own.
(C) is incorrect for the same reason. The argument doesn’t assume that equal exercise will necessarily lead to a healthy back, but simply states that equal muscle strength is one thing that is necessary for a healthy back.
(D) is way too extreme and certainly not an assumption required by the argument.
(E) is not a required assumption either. The author doesn’t necessarily assume that the exercise will be daily exercise. Remember, the answer we choose must be a required assumption.


#officialexplanation
 
mkeat_theraptor
Thanks Received: 4
Forum Guests
 
Posts: 13
Joined: July 08th, 2009
 
 
trophy
First Responder
 

Q17 - When exercising the muscles

by mkeat_theraptor Tue Aug 25, 2009 6:59 pm

Help.
 
stackoutawinner
Thanks Received: 2
Jackie Chiles
Jackie Chiles
 
Posts: 45
Joined: June 30th, 2009
 
 
trophy
First Responder
 

Re: June 2007, S3, Q17 When excercising the muscles

by stackoutawinner Tue Aug 25, 2009 9:00 pm

I'll do my best...

It's important to exercise the muscles on each side of the spine equally if you want to maintain a healthy back.

Balanced muscle development is needed for a healthy back BECAUSE muscles must pull equally in opposite directions to maintain spine alignment and to protect the spine.


You can see the structure of the argument here as I've re-written it. You have a subconclusion in the second sentence premised on the muscles pulling equally in opposite directions. The conclusion is the first sentence and there's a shift in terms... balanced muscle development to EXERCISING the muscles equally. The shift is very subtle, but it's there... is exercise the same thing as development? Not entirely.

What type of missing premise would bridge the gap? Without looking at the answer choices (and I haven't done that yet as I type), I'm looking for something to bridge the gap between development and exercise, but this is the LSAT and they may throw a curve ball at me.

Answer choice A restates the argument. Not a terrible answer, but it seems incomplete to me at first pass. I'm going to defer

Answer choice B seems to be stating the contra-positive of the REQUIRED UNSTATED premise. According to the argument, if we have a healthy spine then we have balanced muscle development (the original argument actually uses the words "needed" which is another term for necessary). If we exercise unequally, will we upset this balance? I don't know... that's what the argument assumes.

I'm now fairly confident this is the correct choice, but to make sure, I'm going to cheat and look....

WHEW... okay, I may have had to delete the post, but now that I know I have it correct, let me discuss it further.

Argument goes like this:
Healthy back requires balanced muscle development.
Exercising equally leads to balanced muscle development. *unstated premise
Therefore, exercising equally is required for healthy back.

When I said that answer choice B appeared to be the contrapositive of the assumption, we have B --> E, and therefore -E --> -B

Did my effort help?
User avatar
 
ManhattanPrepLSAT2
Thanks Received: 311
Atticus Finch
Atticus Finch
 
Posts: 303
Joined: July 14th, 2009
 
This post thanked 1 time.
 
 

Re: June 2007, S3, Q17 When excercising the muscles

by ManhattanPrepLSAT2 Thu Aug 27, 2009 5:39 pm

I'll just add a bit to Stacks' post--

The logic in the original argument can be understood as follows --

exercise both sides equally -> balanced muscle development - > healthy back.

Some of the hardest assumption questions are ones that involve an assumption that is so automatic that it seems unnecessary. Here, it makes perfect sense that you need to exercise both sides equally to promote balanced muscle development...

but maybe that's not true.

(B) exposes this tiny assumption.

If you are unsure it's necessary, think about the reverse: "Exercising the muscles on opposite sides of the spine unequally tends to lead to balanced muscle development."

Notice that the reverse of the assumption would destroy the argument. A good sign that the assumption was necessary.
 
mkeat_theraptor
Thanks Received: 4
Forum Guests
 
Posts: 13
Joined: July 08th, 2009
 
This post thanked 1 time.
 
trophy
First Responder
 

Re: Q17 - When exercising the muscles

by mkeat_theraptor Sat Aug 29, 2009 11:52 am

Thank you, both are helpful. Although to be honest, I think in an actual test situation this is a question I would get wrong again due to the subtleness of the language.

Agreeing with Stack's diagramming,

Premise: Maintain healthy back -----> balanced muscle development
Assumption: Balanced muscle development -----> exercise muscles equally
Conclusion: Maintain healthy back -----> exercise muscles equally

This seems very straightforward with a diagram. With such, I think it's also helpful to use the eliminate wrong answer strategy and probably the strategy I would rely on in an actual test situation, where I might have difficulty quickly elucidating a diagram .

A) Balanced muscle development is a necessary, not sufficient condition. Eliminate.
B) Seems to link balanced muscle development and exercise muscles equally. Keep.
C) Exercise muscles equally is a necessary, not sufficient condition. Eliminate.
D) Irreparably damaged. Too strong/Out of scope. Eliminate.
E) Daily. Never mentioned/Out of scope. Eliminate.

So that only leaves B.
 
tianfeng102
Thanks Received: 11
Forum Guests
 
Posts: 21
Joined: August 23rd, 2010
 
This post thanked 1 time.
 
 

Re: June 07, S3, Q17 When exercising the muscles in one's

by tianfeng102 Sun Nov 28, 2010 6:02 pm

In terms of formal logic, the first two sentences can be "coded" as the followings:

#1 When exercising the muscles in one’s back, it is important, in order to maintain a healthy back, to exercise the muscles on opposite sides of the spine equally.
Maintain a healthy back --> Exercise muscles equally
Contrapositive: NOT exercise muscles equally --> NOT Maintain a healthy back

#2 After all, balanced muscle development is needed to maintain a healthy back . . .
Maintain a healthy back --> Balanced muscle development
contrapositive: NOT Balanced muscle development --> NOT Maintain a healthy back

In addition, it is obvious that #1 sentence is the conclusion and #2 sentence is the premise.

Then based on the premise of "NOT Balanced muscle development --> NOT Maintain a healthy back", for the logic chain moving from "NOT exercise muscles equally" to "NOT Maintain a healthy back", it has to have this bridge "NOT exercise muscles equally --> NOT Balanced muscle development". Answer B describes this bridge in typical LSAT fashion.
LSAT could change from demon to darling, if you tame the beast (PrepTest) one after another in 60 days.
User avatar
 
tamwaiman
Thanks Received: 26
Forum Guests
 
Posts: 142
Joined: April 21st, 2010
 
 
trophy
Most Thankful
 

Re: June 07, S3, Q17 - When exercising the muscles

by tamwaiman Thu Jun 02, 2011 11:48 pm

I wonder whether my understanding is wrong. Can someone help to check my reasoning? Thanks.


I divided this argument into three parts, a main conclusion, an intermediate conclusion and an evidences as follows:

MHB --> BMD

KBPS --> EOE
----------------------
MHB --> EOE

(MHB: maintain a healthy back
BMD: balanced muscle development
KBPS: keep the back in proper alignment and protect the spine
EOE: exercise on opposite sides equally)

The gap is between BMD & KBPS, so I look for an answer such as "To balance muscle development one must keep the back in proper alignment and protect the spine." To my surprise, instead of my guess, the answer neglect the evidence and connect BMD & EOE. (However, I still can get the answer by eliminating the other wrong answers)
 
zainrizvi
Thanks Received: 16
Atticus Finch
Atticus Finch
 
Posts: 171
Joined: July 19th, 2011
 
 
trophy
First Responder
 

Re: June 07, S3, Q17 - When exercising the muscles

by zainrizvi Thu Dec 01, 2011 2:51 pm

This is quite a tricky answer choice.

(B) to me seems out of scope. I understand the big gap in the argument in the present that there is no relationship given between exercising muscles equally and balanced muscle development. So I thought an answer choice would be equal muscle development would lead to balanced muscle development. I guess to add a layer of complexity - they talked about exercising unequally. Since logically speaking there are only two options (equally or not equally), and the only reason given for exercising equally is balanced muscle development. Well, what if balanced muscle development could be achieved through unequal exercising as well. Then the basis for the argument - the basis for choosing between the two options - is gone.
 
goriano
Thanks Received: 12
Atticus Finch
Atticus Finch
 
Posts: 113
Joined: December 03rd, 2011
 
 
 

Re: Q17 - When exercising the muscles

by goriano Tue Mar 13, 2012 2:51 pm

zainrizvi Wrote:This is quite a tricky answer choice.

(B) to me seems out of scope. I understand the big gap in the argument in the present that there is no relationship given between exercising muscles equally and balanced muscle development. So I thought an answer choice would be equal muscle development would lead to balanced muscle development. I guess to add a layer of complexity - they talked about exercising unequally. Since logically speaking there are only two options (equally or not equally), and the only reason given for exercising equally is balanced muscle development. Well, what if balanced muscle development could be achieved through unequal exercising as well. Then the basis for the argument - the basis for choosing between the two options - is gone.


I don't think that's right. The correct answer choice should be the reverse of what you stated, so "Balanced muscle development leads to exercising equally." The contrapositive of this statement is expressed in (B)
User avatar
 
ohthatpatrick
Thanks Received: 3808
Atticus Finch
Atticus Finch
 
Posts: 4661
Joined: April 01st, 2011
 
 
 

Re: Q17 - When exercising the muscles

by ohthatpatrick Wed Mar 21, 2012 3:38 pm

I'm not sure I understood Zainrizvi's post. It seemed like he/she had some hesitation about (B), yet the final objection he/she raised, "what if exercising unequally could lead to balanced muscle development as well", is exactly right and that's precisely why (B) IS the correct answer.

Objections and assumptions are two sides of the same coin.

If your objection is
"Exercising unequally could still lead to balanced muscle development"
then the corresponding assumption being made is
"Exercising unequally could NOT still lead to balanced muscle development".

That is what (B) is giving us, notwithstanding the slightly different strength of wording. If you negate (B), you get the objection you had in mind (and that's how you know (B) is correct).

The other two posters who broke it down more symbolically were both right-on, mostly. The girl who found the subsidiary conclusion and anticipated a link between what seemed to be the dangling threads of the argument was mainly correct in her thinking, although she was allowing EOE to mean both "exercise the muscles equally" and "the muscles must pull equally in opposing directions". Those do not mean the same thing as each other, so we shouldn't try equating them under one symbol.

Because of that, there isn't really a way to involve that last clause of the argument as a symbolic match for anything mentioned previously. Nothing following the "since" is equivalent to any of the previous concepts introduced.

Let me know if anyone has lingering questions on this one.
 
hornswaggle
Thanks Received: 0
Forum Guests
 
Posts: 6
Joined: February 18th, 2013
 
 
 

Re: Q17 - When exercising the muscles

by hornswaggle Wed May 22, 2013 10:19 pm

I have a few questions about... this question.

The stem contains two sentences that appear to have conditional logic:

1) In order to have a healthy back, it is important to exercise both sides equally (HB->Exercise!)
2) In order to have a healthy back, you need balanced muscle development (HB->BMD),

It argues that HB->Exercise! because HB->BMD.

My first question: is 1) a strict conditional relationship? Or does the qualifier "important" make it not so?

I picked B) because it was the only answer choice that addressed the gap between Exercise! and balanced muscle development. It states that without Exercise!, you tend to not have bmd.

My second question: what would the answer look like if this were a sufficient assumption question? Would it be "without Exercise!, you cannot have bmd?" Or, would "If you exercise!, you will have bmd?" be valid as well?

I greatly appreciate any help whatsoever.
User avatar
 
ManhattanPrepLSAT1
Thanks Received: 1909
Atticus Finch
Atticus Finch
 
Posts: 2851
Joined: October 07th, 2009
 
 
 

Re: Q17 - When exercising the muscles

by ManhattanPrepLSAT1 Fri May 24, 2013 4:17 pm

Hi hornswaggle, good questions.

I see the argument broken into three pieces: main conclusion, subsidiary conclusion, premise. The terms in the premise are tough to see how the can be easily deemed similar to the terms in the main conclusion or subsidiary conclusion. So, I'd focus my energy on the gap between the subsidiary conclusion and the main conclusion.

HB --> BMD
--------------
HB --> EOSE

(Notation Key: HB - healthy back, BMD - balanced muscle development, EOSE - exercise opposite sides equally)

The gap is BMD --> EOSE, which is stated in answer choice (B) in the form of a contrapositive.

Your question about HB --> BMD and whether that is a strict conditional - it's not a strict conditional, but close enough to one that I'd write it that way.

To your second question about what the answer would look like if this were a sufficient assumption question - honestly, it'd look nearly the same. When you find yourself linking a string of conditional relationships, whether you're on a necessary or sufficient assumption, your task is simply to connect the terms. In a sufficient assumption question, the language might be stronger, but the idea of connecting the dots is where you'd really put your focus.

Hope that helps!
 
Amontillado
Thanks Received: 0
Vinny Gambini
Vinny Gambini
 
Posts: 7
Joined: August 03rd, 2014
 
 
 

Re: Q17 - When exercising the muscles

by Amontillado Tue Aug 19, 2014 12:36 am

Hi, for this question, do we kind of ignore the words after "since" at the end? I think that should be another (necessary) assumption required by the argument, right?

Premise: PAPS-->EOSE Intermediate Conclusion: MHB-->BMD Final Conclusion: MHB-->EOSE

We are bridging the gap from intermediate to final conclusion in this question. What about the first gap? Or, is it really a gap?

If this were a sufficient assumption question, do we need to consider that as well?

Thank you : )
 
BensonC202
Thanks Received: 0
Vinny Gambini
Vinny Gambini
 
Posts: 19
Joined: April 08th, 2019
 
 
 

Re: Q17 - When exercising the muscles

by BensonC202 Sat Jul 18, 2020 7:01 pm

dan Wrote:17. (B)
Question Type: Assumption

The argument seems reasonable, but consider someone who may, for whatever reason, have unequal strength on either side of his spine to begin with. Wouldn’t he want to exercise the weaker side more than the stronger side in order to make them equal? The assumption in the argument is that unequal exercise leads to unequal muscle strength (in fact this may not be true for all people). Answer (B) clearly expresses this assumption.

(A) is incorrect. The argument does not need to assume that equal muscle strength will be enough on its own to keep the spine aligned (other things might be necessary as well). The argument says that equal muscle strength is one thing that is required to keep the spine aligned, but it doesn’t have to be enough on its own.
(C) is incorrect for the same reason. The argument doesn’t assume that equal exercise will necessarily lead to a healthy back, but simply states that equal muscle strength is one thing that is necessary for a healthy back.
(D) is way too extreme and certainly not an assumption required by the argument.
(E) is not a required assumption either. The author doesn’t necessarily assume that the exercise will be daily exercise. Remember, the answer we choose must be a required assumption.


#officialexplanation



Allow me to elaborate Answers with my own reasoning.

Apparently,

Both sides of the muscle be pulled equally is necessary to keep the back in proper alignment and protect the spine is the support offered to lead to the sub-conclusion that if maintain healthy back, balanced muscle development is required and by which to support the final conclusion that it is required to exercise the spine equally if to maintain a healthy back.

1. Proper alignment + protect the spine -> Both side of the muscle be equally pull
2. Maintain healthy back -> balanced muscle development is required

C. To maintain a healthy back -> exercise spine equally.

Answer A is the mistaken reverse on the premise 1

Answer B is the contrapositive from the inferred logical chain, by combining premise2 and conclusion, that Balanced muscle development ---> exercise spine equally.

Unequally exercising muscles ---> unbalanced muscle development. Correct Answer.

Answer C is also the mistaken reverse based on the argument.

Answer D says that unequally side of the spine are exercised unequally ---> one's back will be irreparably damaged.

Not irreparably damaged does not equate to maintain a healthy back ( something is not broken does not mean its definitely in good condition )

Answer E is not required.